Showing posts with label logical reasoning. Show all posts
Showing posts with label logical reasoning. Show all posts

Sufficient Assumption Questions and the Negation Test

LSAT Blog Sufficient Assumption Questions Negation TestIn this LSAT Blog post, I discuss two common Logical Reasoning question types: Necessary Assumption and Sufficient Assumption.

The negation test is a common strategy you can effectively use for Necessary Assumption questions. However, many test-takers like the technique so much, they try to use it for sufficient assumption questions also, often leading to disastrous consequences.

Due to the difference between Necessary Assumption and Sufficient Assumption questions, you shouldn't use it for sufficient assumption questions.

The reason is that a necessary assumption always needs to be the case in order for an argument to work (the conclusion follow), but a sufficient assumption is just that - sufficient to make the argument work (the conclusion follow), but not all sufficient assumptions are required in order for the argument to work (for the conclusion to follow).

In other words, the correct answer to sufficient assumption questions is often something that does not actually need to be true, but if it is true, will fully justify the conclusion.

(Note that many answers to sufficient assumption questions are broader than the argument.)

***

I'll illustrate this with two examples:

The first is a very general and simple example , but the second is an example from an actual LSAT PrepTest.


General example:

Take this statement:

"When your dog encountered my cat in the yard, your dog attacked my cat."

It would be sufficient to prove this statement if we learned, as new information, that when any dog encounters any cat, the dog will attack the cat.

However, this doesn't *need* to be the case in order to prove the statement above. Perhaps some dogs have no problem with cats, but this particular dog was getting back at the cat for being a total jerk.



Example from an actual LSAT PrepTest:

Please see PrepTest 35 (Oct 2001), Section 1, Question 22 (p226 in Next 10).

We can diagram the stimulus as:
Evidence: C -> NOT T -> NOT P
Evidence: P -> T -> NOT C
---
Conclusion: P -> NOT H
While H -> C is sufficient to guarantee the conclusion, it's not required in order to do so. Even if H didn't require C, information from the evidence could still, in combination with a sufficient assumption, guarantee that P required NOT H.

What if we learned that T required NOT H, or that H required NOT T? In combination with the statement that P -> T, the conclusion would still follow from the evidence (even if H didn't require C).

(It's possible that LSAC would disregard the T -> NOT C statement when considering how to make the evidence + sufficient assumption lead to the conclusion - stimuli often contain information that serves as filler and is irrelevant in guaranteeing the conclusion.)


Photo by jjpacres


LSAT Logic Prep: Jay Z's Blueprint vs. Occupy All Streets

LSAT Blog Blueprint LSAT Prep Logic OccupyIn this LSAT Blog post, I cover Jay Z's approach to charitable giving in two different situations, giving an example of a principle that might underlie each.

Why?

Because the LSAT Logical Reasoning section has increased the number of principle questions in recent years.

The LSAT more frequently asks you to take a given principle and apply it to different situations.

***

On 9/11, Jay Z dropped his sixth album, The Blueprint. He toured during the months of September and October 2001, donating a dollar from every concert ticket sold to 9/11 relief organizations.

This week, Jay Z's Rocawear clothing line will release a line of T-shirts with the message "Occupy All Streets." His spokesperson stated:
The 'Occupy All Streets' T shirt was created in support of the 'Occupy Wall Street' movement. Rocawear strongly encourages all forms of constructive expression, whether it be artistic, political or social. 'Occupy All Streets' is our way of reminding people that there is change to be made everywhere, not just on Wall Street. At this time we have not made an official commitment to monetarily support the movement.

(Although they were briefly removed from Rocawear.com, they're back, should you be interested.)

***

So, what principles might help to justify Jay Z's donations (or lack thereof) in each situation?


For Jay Z's concerts surrounding the release of The Blueprint:

Principle: "If an artist drops an album on the same date as that of a national crisis, the artist should make some financial contribution to the victims of that crisis."

Application: Jay Z dropped The Blueprint on 9/11, and he made donations to 9/11 victims.



For Jay Z's new T-shirts:

Principle: "Even if an artist makes a profit from the sale of merchandise related to a political movement, this does not obligate to make a financial contribution to that movement."

Application: Jay Z makes money from selling "Occupy All Streets" T-shirts, but he doesn't give any money to Occupy Wall Street.


For both:

Principle: "Supporting those affected by political or economic events does not constitute a requirement to provide financial support for those affected."

Application: Jay Z supports both 9/11 victims and Occupy Wall Street, yet gives money to the former, but not the latter.


LSAT Logic and Libertarian Seasteading

LSAT Logic and Libertarian SeasteadingLSAT Blog reader Jason wrote the following LSAT-style analysis of a real-world situation. Please share your questions for him, and thank him for sharing his analysis, in the comments!

***

I read an article on "libertarian seasteading," oil-rig-like platforms floating in sovereign waters outside the reach of -- and free and secure from -- partisan politics, excessive taxes and politically-inspired moralities.

The argument in favor of seasteading:

-assumes (depends upon/takes for granted) that a practically defenseless "country" is able to remain sovereign for any reasonable amount of time.

-could be strengthened by adding that such libertarian nations plan to fortify, significantly, by hiring private navies to patrol international waters surrounding their "seasteads."

-could be weakened by the fact that piracy on the high seas has been on the rise since the end of the Cold War and subsequent decreased emphasis on naval presence by both the U.S. and Soviet navies.

-could be weakened by the fact that since "seasteads" cannot be entirely self-sufficient, they must be located close enough -- within 50 miles -- to traditional nation-states for service of consumables. That "seasteads" have a tendency to drift more than 50 miles at a given time. That, of course, assumes that the "seasteads" will drift into the direction of the neighboring nation-state's sovereign zone and not away from it. And, that the neighboring states are not libertarian. And, that a temporary trespass of a "seastead" would be treated like a ship entering a nation-state's zone of influence rather than a unique event -- drift of a nation-state towards another. At that point, for example, in the absence of prior custom, would the permanent nation-state's influence supersede the "seastead," or vice versa?

Of course, the possibilities for trap answers (citing a timeline on the drift and distance, etc) and other logical fallacies are endless.

To spice up the answer choices, a "blogger" could be cited as a source. For example, in this case, "Joe Opinion from 'alwaysperfectlycorrect.web' (I felt tempted to write something like 'alwaysright.web,' but that certainly violates LSAC's directive to try and keep things unemotional) says that this is a bad idea because it's just not practical." Or, "It's a terrible idea, a country free from moral obligation will likely be an outpost for prostitution and drugs."

Then we work in an appeal to (non-credible) authority, appeal to emotion, appeal to a certain moral directive. And, of course, improper prediction of inevitability.

(Come to think of it, I haven't seen a "blogger" cited in any of the questions I've yet studied. As my background has been the advertising industry, I love the logic flaws presented in the "Advertisement" prompts.)

And, of course, we could focus the question as the 2-speaker, counterargument style:

CRITIC: "Ultimately, it won't achieve it's goal of lower taxes because the out-of-pocket costs to ensure the safety and security of the 'seastead' essentially amounts to a tax."

In LSAT language, this changes the meaning of a key term.

LSAT Logic, the Environment, and Vegetarianism

LSAT Blog Logic Environment VegetarianismLSAT Blog reader Emily wrote the following LSAT-style analysis of a real-world situation. Please share your questions for her, and thank her for sharing her analysis, in the comments!


***

I based the following on an article titled: Becoming Vegetarian Can Harm The Environment.

The study in question claims that "A switch from beef and milk to highly refined livestock product analogues such as tofu could actually increase the quantity of arable land needed to supply the UK." This statement is not entirely well summarized by the title of the article, "Becoming vegetarian 'can harm the environment". After reading through the clip, the limitations of the study become clear, and the notion that vegetarianism is more detrimental to the environment than eating as an omnivore appears more and more fallacious.

The important question that this article highlights, and that is probably what draws most people to read it after noticing the title, is about whether modern vegetarianism is actually good for the environment or not. Can vegetarianism harm the environment? Despite leading us to believe otherwise, that is not a question that can be answered by this study. Several dubious assumptions are implicitly made in drawing this specific title from the research that is presented.

For one, the study deals exclusively with British interests. Collins claims that eating processed analogues can harm the environment, yet from what we read we see that the research is very particular to qualities of the United Kingdom. The study compares the pro's and con's of raising livestock for meat in Britain, and in importing processed vegetarian alternatives. Given that Britain is an island, the environmental "con" of importing goods is much weightier than it would probably be in most other countries. It is thus misleading to generalize geographically about the environmental costs of importing food.

It is also wrong to generalize based on the idea that meat is produced more responsibly domestically. If Collins is to claim that replacing meat with tofu can be environmentally irresponsible, he should note that if you live in a country where farming practices are poor (unlike the United Kingdom) then the reverse may be true.

But neither of these ideas would really be relevant in the end, because it makes no sense for meat production to shift to less regulated countries when more people become vegetarian. If the British farming industry suffered, it would be because demand was reduced, not because people were buying more imported meat. In addition, there is no explanation for why crops like soybeans would have to be produced abroad rather than at home, were the number of vegetarians to increase in Britain.

Extending this initial claim beyond the borders of Britain shows a myriad of other discrepancies in logic. It would be one thing for the whole population of Britain to become vegetarian. Perhaps more land would be cleared elsewhere for other crops if Britain could not sustain its own tofu industry.

Yet were the logic to be applied on a global scale, the results would be different. There is no clear comparison of how many acres of land it takes to produce a unit of tofu compared to a unit of beef, but it seems unlikely that beef would require less land. Therefore, the claim that replacing meat with meat analogues uses more land in general may be wrong. Additionally, if meat eaters around the world were replaced with vegetarians, there is no evidence here that the net amount of transportation required for food distribution would increase. Presumably food would need to travel from one place to another either way, even if those pathways were sometimes reversed or shifted.

Lastly and most crucially, Collins makes the assumption that transitioning from eating meat to not eating meat means replacing animal products with processed analogues on a 1:1 basis. He does not explain whether the trade-off is made by weight, volume, calorie, etc., but regardless of the conversion factor it is a faulty assumption.

Nowhere in this research, to my knowledge, does it say that residents of the United Kingdom eat an equivalent amount of analogue products to their previous animal consumption when they become vegetarian. Based on the study, the ONLY way that vegetarianism could potentially harm the environment is if people consume products like tofu at a rate equal to or higher than their past meat consumption, and this seems to be the most far-fetched assumption yet made.

The research is presented by Collins in a way that first traps the eye and leads one to think that broader and better substantiated claims are being made in what is about to be discussed. Unfortunately, while the research itself appears sound, it is not the wide-sweeping blow to vegetarianism that some may have hoped for. Interesting study, but extremely difficult to generalize from.

Photo by texhex

LSAT Practice: Games and Reasoning Workbook

Whether you've finally run out of actual LSAT PrepTests, or you just want to review the basics as Test Day approaches, I've got a book for you: LSAT Practice: Games and Reasoning Workbook.

You may need some more practice, or you may just want some drills and exercises to help you reinforce the strategies you've learned.

Fortunately, fellow LSAT tutor Jesse wrote this workbook to test your understanding of basic LSAT concepts for the Logic Games and Logical Reasoning sections. And it'll help you review strategies and approaches for each.

Each section of the workbook includes a general review of the topic covered, followed by drills and exercises on that topic to test and reinforce your understanding.

Some of the topics the workbook covers:
  • General approaches and strategies for Logic Games and diagramming
  • General approaches and strategies for Logical Reasoning questions
  • Necessary and sufficient conditions
  • The contrapositive
  • Negation
  • Logical quantities (some, many, most, all, and none)


Get your copy for only $19.97:

CC



Fun Facts:

-This workbook is 82 pages, and you get it sent to your email immediately after purchasing as a PDF. This means you can print new copies of the exercises and drills to redo them for extra practice.

-The instant download link will be sent to whichever email address you submit.


Win Free LSAT Books: A Contest

LSAT Blog Win Free LSAT Books ContestWant free LSAT books? Of course you do.

I'm running a contest on the blog for the next week.

It's simple - you email me an LSAT-style analysis of a news article, online video, etc. You can find examples in my blog posts that analyze logic in real life.

If yours is one of the best 5 submissions, you'll win a free LSAT book, or two, or three.

The prizes:

One 1st place winner will receive a free copy of each of the 3 More GROUPED books: More GROUPED by Game Type, More GROUPED by Question Type, and More GROUPED by Passage Type (value: $160).

One 2nd place winner will receive 2 of the 3 More GROUPED books listed above (value: $85-$120).

Three 3rd place winners will receive a copy of More GROUPED by Game Type or GROUPED by Game Type (value: $35-$40).

Infinite 4th place winners can win eternal Internet fame by having their submissions published on the blog.

The deadline for submissions is Wednesday, August 31, 11:59PM Eastern Time.

Please email them to me in the body of the email, not as attachments.

Good luck, and I look forward to reading your submissions!


***

I've included below some suggestions of news articles that you might analyze. However, feel free to submit an analysis of any publicly-accessible article or video you find online.

Again, you can find examples in my blog posts that analyze logic in real life.


Articles


People argue just to win, scholars assert


That morning smoke is probably going to kill you


Law schools' women problem

Do Cellphones Cause Brain Cancer?

Justice is served, but more so after lunch: how food-breaks sway the decisions of judges

Study: Women Post More Facebook Photos to Raise Self-Esteem

Facebook Use Linked To Lower Grades In College

Freedom to surf: workers more productive if allowed to use the internet for leisure

Organic food is no healthier, study finds

Becoming vegetarian 'can harm the environment'


Want a Higher G.P.A.? Go to a Private College


Why Liberals and Atheists Are More Intelligent


Firm Body, No Workout Required?


For Today’s Graduate, Just One Word: Statistics


Standardized Test Scores: Voodoo Statistics


The Debate Is Still On: The Mysterious Downfall of the Neanderthals



***

By submitting an entry, you are granting me the right to edit and publish your submission on the blog.

(When publishing submissions, I'm happy to use your first name only, initials, a pseudonym, or to keep it completely anonymous - your choice.)

Photo by aresauburnphotos

LSAT Logic and the New Cigarette Warning Labels

LSAT Blog Logic FDA Cigarette Warning LabelsLSAT Blog "Reader-of-the-Decade" Caleb already wrote an LSAT diary and a Logic Game for all of us.

He's back, and this time he's got an LSAT-style analysis of the reasoning behind the upcoming cigarette warning labels required by the FDA (PDF).

Thanks for Caleb for sharing his thoughts, and be sure to leave some comments letting him know what you think of his arguments.

Caleb's analysis:

If the LSAT gods smiled kindly upon you (they won’t) and said that an entire logical reasoning section was going to be based on the new Family Smoking Prevention and Tobacco Control Act, where would you start your studying?

Since you are a LSAT study-monster with aspirations of lawyerin’, this trip to fantasy land (my second home) could be beneficial in your preparation. After all, a key to crushing the LSAT is the ability to objectively analyze arguments without your personal opinions on the subject interfering with your ability to reason. And lots of people have strong views on smoking, right? “Eww! Smoke stinks and stays in my hair for days!” and “I do what I want” are two key opinions that come to mind.

First, let’s get a hold of the basic argument. You can read the text of the bill here (which I recommend), but since you’re lazy busy practicing for the games section I’ll try and give an objective summary. Then, we’ll take a look at pieces of the argument and try to bend and twist them to our will. And, since you will encounter varying levels of difficulty on LSAT questions, we’ll try and get as fiendish and dastardly clever as possible. Because you know the sadistic hobgoblins at the LSAT factory surely will. So let’s start with a summary:
Smoking is bad, mmkay. Tobacco causes damage both in terms of human health and economic impact. Tobacco use costs the US billions of dollars in healthcare and lost productivity. Tobacco companies target young smokers with their advertising. Nicotine is addictive and people who want to quit using it find it very difficult to do so. Lawmakers have an obligation and mandate to protect citizens- especially children- and should oversee and regulate the tobacco industry. An effective strategy to lower the number of smokers and stop underage children from starting is to place graphic images and warnings on tobacco products and advertising, and to ban any flavored cigarettes. Except menthols [true story].
I trained like a Jedi master for the LSAT- with Steve’s LSAT Blog as my lightsaber, of course- so I can’t even type that summary without my brain pumping out fifty different questions and counter-arguments that all bottleneck somewhere in my Broca’s Area (look it up, lazy). I’ll just step out for a second, have a smoke, and come back so we can start in!

Ahh, that’s the stuff.

Now, let’s start with the immortal words of me:

“All reasoning starts with questioning.” –Caleb Shreves

You’re welcome. Here are a few obvious questions that popped into my head while typing that summary:

1. Why is there an economic impact from healthcare and lost productivity?
2. Is tobacco advertising responsible for people smoking? If so, to what degree?
3. How are tobacco companies targeting minors?
4. Why is regulation and, specifically, using graphic warnings an effective deterrent to smoking?
5. Why should legal adult practices be regulated by Government?
6. Why does banning flavored tobacco lower smoking rates?
7. How is “lost productivity” defined and measured?

Add a few of your own. Go ahead, I’ll wait. Now let’s take some of the questions I posed and think of how an LSAT test-maker might think to incorporate them. Here are some easy ideas:

Strengthen: Other countries have implemented similar policies and found them to be effective in deterring tobacco use.

Necessary Assumption: Flavored cigarettes are attractive to minors

Conclusion (inference): The fewer minors who start smoking, the more money the country will save.

Try a few on your own! You should be able to come up with quite a few of these easy ones pretty quickly, and you’ll likely face a few test questions built off these types of low-hanging fruit. But let’s take it up a notch and expand some of these arguments to make them a little more complex. Here’s something based off my dad’s cynical argument, which I call “dying is cheaper.”

End of life care is, by far, the most expensive piece of the healthcare pie. Tobacco users die, on average, ten years sooner than non-users. Thus, tobacco users are doing everyone else a favor.

Great work, dad. Any questions spring to mind when you read that argument? How about a sample LSAT setup to help you out:

Dr. A: “Healthcare costs should be a primary concern when dealing with tobacco regulation. Hence, we should find ways to reduce the number of smokers.”

Dr. B: “I agree that healthcare costs are a primary concern, which is why I encourage more people to start smoking.”

See any conflicting assumptions here? This could be the start of a “the two doctors disagree about…” type question, or a “which statement, if true, would lend support to Dr. A’s conclusion…” type question. Now that we’re this far, let’s scramble the answer up, LSAT style, and get tricksy. Consider which Dr. would be validated by the following evidence:

“Recent studies have shown that treatments for certain common long-term illnesses requiring regular hospital care increase in total healthcare cost at an exponential rate as the person with the illness ages.”

Tricksy indeed! Read the setup again; the doctors are disagreeing about whether the costs of a tobacco-related death outweigh the costs of living an additional 10 years, tobacco-free. If common illnesses cost a crap-ton more every year as you age, then people dying early would, in fact, probably save money! This would be good for Dr. B (surprise- that was dad’s side of the argument!). But you can see how a few questions about an argument, combined with some assumptions, and sprinkled with fuzzy language can give you a pretty nasty LSAT question.

Finally, let’s work together and come up with something really nasty. How about we start with the FDA’s argument about minors:

A primary reason that minors begin using tobacco is the prevalence of advertising they are exposed to. Thus, we should require that tobacco advertisements on billboards and posters contain gruesome and graphic pictures of the consequences of tobacco use.

Allllllrighty then, FDA. What are the assumptions here? Well, first we would assume that minors actually see tobacco advertising, right? Do they? We would need to assume that the advertising that minors see actually affects whether or not they use tobacco wouldn’t we? Why would graphic images stop a minor from smoking? Don’t minors see graphic images on TV and in video games every day? What if studies showed that minors actually become desensitized by graphic imagery and were more likely to use tobacco if this policy were in place? Asking questions like this can immediately help point out the assumptions of any argument- and, accordingly, help you determine how an LSAT question might be framed by them.

Fun (and true) fact: tobacco brands that are heavily-advertised are much more likely to be used by minors than adults. But what if this were reversed? If minors were less likely than adults to smoke heavily-advertised brands, wouldn’t that mean that the advertising doesn’t affect them? (if you’re questioning even this, then you are well on your way to true LSAT dominance). Going with this theme, I would expect a test answer to include a convoluted piece of evidence that showed a link between advertised brands and what minors actually smoked. For starters, we could weaken the FDA’s argument with a statement like:

“A recent study of smokers aged 12-16 found them to have a distinct preference for Brand X cigarettes, a brand that advertises its products far less than its area competitors.”

Okay, we can grasp that. Kids smoke even without the advertising, so advertising ain’t a big deal like the FDA said. But let’s step out farther and imagine some of the fiendish tricks used by the LSAT hobgoblins. How would the LSAT refute this last statement?

“Legislation passed in the last year has forced Brand X to dramatically reduce its advertising budget.”

Now hold on a minute. Maybe these kids had already been exposed to Brand X! Timeline trickery, that is (and one that the LSAT uses sometimes, trust me). Imagine a “which answer, if true, would provide the LEAST support for…” type question on this, using evidence to prove something that is counter to what’s true in the real world, couching it in fuzzy language, and throwing a trick answer in to boot. You can see how the LSAT can take simple things and make them vastly more complicated. Reminds me of my girlfriend.

What’s the point of all this hypothetical nonsense you ask? The more you can quickly and automatically pre-form arguments and assumptions in your mind, the better you’ll do on test day. If every day you take an issue like this and spend 10-15 minutes thinking of how you would create test questions, you will make neural pathways in your brain that will give you X-ray logic-vision when reading new and unfamiliar arguments.

And that sounds good, right? Plus, you might just start seeing the issues you encounter in daily life in a new light, too. Like smoking. Whatever your views on smoking happen to be, I challenge you to look critically at the reasoning behind the decisions made by our Government on this issue. If you can come away saying “nope, looks like they’re air-tight!” then good luck on test day. But I would wager my Xbox and all my Halo games that you’ll at least be a bit more skeptical of these policies and the true rationale behind them if you take the time to examine them. And I don’t wager my Halo-goodness lightly, either.

Caleb

P.S. While I encourage you read the full text of H.R. 1256, I can’t resist showing you item #4 in the “findings” section of the bill:

“Virtually all new users of tobacco products are under the minimum legal age to purchase such products.”

I started smoking at age 26. Should I call my senator?

Sample Logical Reasoning Questions

LSAT Blog Sample Logical Reasoning QuestionsBlog reader Zach recently wrote to me with a few sample Logical Reasoning questions he's written himself. Although real LSAT questions are best to practice on, these are also good practice and completely free to share with all of you!

Here's what Zach wrote to me:

With the LSAT coming up, my mind has been focused in just one area recently. Everything I do, see or hear is relevant to this test and this (faked) conversation between my mom and the dog is no different.

I wrote a couple of sample Logical Reasoning questions below based on their conversation.

Here they are:

Mama: You better poop when I take you outside. If you don't poop outside, I will make you stay on the bed all day. Therefore, you cannot roam around on the floor.

The dog: Your reasoning is flawed. I can jump off the bed to the floor anyway, regardless if I poop outside. Therefore, me pooping outside does not have any impact on my freedom.

1. Which one of the following is most parallel to Mama's reasoning?

(A) You must eat carrots when we go to this restaurant. If you do eat carrots, you will gain better eyesight. Therefore, you might be able to read better.

(B) You should get gas when we pass Springfield. If you don't get gas, we will stall out near Huntsville. Therefore, we might be murdered.

(C) You better have a drink when you go the bars. If you don't have a drink, you won't have a lot of fun. Therefore, you won't have good stories to tell.

(D) You better try these mushrooms when we go into the magical forest. If you do try them, you will see magical creatures. Therefore, you cannot disbelieve their existence.

(E) You must be quiet when we go to Amishville. If you are not quiet, you will be shunned. Therefore, you might embarrass me.


2. Which one of the following most weakens the dog's argument?

(A) The bed is only a few inches off the floor.

(B) The bed is situated only on a twin-sized frame.

(C) The weight remaining within the dog because of his lack of poop is insignificant to his jumping ability.

(D) The dog's concept of freedom is contingent on the amount of digestive material within his larger intestine.

(E) If the dog does not poop outside, Mama puts up an electric fence around the bed barring any entity from accessing or leaving it.


Answers:
(Please highlight the text below to read them, since they are in a white font color.)

1. C
2. E

***

Please let us know your thoughts on these, and how you do on them, in the comments!


LSAT Logic and Law School Spam

LSAT Blog Logic Law School SpamI hate spam. I even did a blog post making fun of spammers.

I publish LSAT Blog on a schedule - once per week - this means you'll only receive one email from me per week - and that's only if you've chosen to subscribe via email.

The emails with new blog posts are sent out automatically. I don't do anything at all with them, and I have trouble just reading all the emails I receive, let alone responding to as many of them as I can.

I really don't have the time or interest to send extra emails or to create a master email list to sell or cackle over in my free time.

So, I was pretty surprised to receive the following email from a blog reader whom I'll call "Jim."



 Here's the text in case you can't see the image:
You clearly sold (or gave) my email address to a law school -- John Marshall. I am extremely careful with this email address and don't get spam on it. I have never received an email from a law school institution and received one from John Marshall within two hours of making a purchase on your blog. I trusted your service to keep my email address confidential.

Sincerely,

"Jim"
In all seriousness, I love receiving accusatory emails from blog readers - it really makes my day.

I generally find that the more accusatory the tone is, the more likely the reader is to be incorrect. In the past, these emails have typically been related to one of my original Logic Games. In these emails, the reader assumes there's a flaw in the game itself, when the flaw is in the reader's own understanding.

However, "Jim's" email is related to an action I had supposedly performed. He accuses me not of incorrect logic, but of purposely doing something super-sketchy.

Since I see much of what goes on in the real world as instructional opportunities, I decided to point out the flawed assumptions in "Jim's" reasoning - for "Jim" himself and for all LSAT Blog readers.

I responded with the following:
Hi "Jim,"

Please do not assume that I gave or sold your email address to anyone. I would never do any such thing.

It would be a remarkable turnaround time for me to take your email address, forward it to John Marshall, then have them send you an email, all within 2 hours of your purchase.

It is common practice for LSAC (to which you have presumably given your email address at some point) to provide certain law schools with applicants' email addresses. This is almost certainly what happened.

A better approach to determine what happened might have been to ask John Marshall how they received your email address before accusing me of having shared it. Had you done so, you might have received something like the following:
We obtained your name and contact information lawfully from the Law School Admission Council, and we have used it in accordance with their Candidate Referral Service (CRS) policies. My suggestion to you is to update your LSAC profile to indicate that you no longer wish to be listed on the CRS – this will save you from receiving messages like this one.
If you don't believe me, why not reply to the email from John Marshall and ask?

I wish you all the best with your LSAT prep. Please see the following link: http://en.wikipedia.org/wiki/Post_hoc_ergo_propter_hoc

Best regards,
Steve
I could've gone on and on in my response about post hoc fallacies with examples from specific PrepTests, but I'd rather let all of you weigh and measure the evidence and various considerations in the comments section of this blog post.

Since "Jim" was kind enough to send me an email acknowledging his mistake, I'll let him have the final words on this post:
Hi Steve,

If I mistakenly accused you sharing my email address, I apologize. As soon as I sent the email, I began to picture your Finding the Flaw in My Reasoning. I don't think it's a pure post hoc fallacy case, however, as there were several conditions increasing the likelihood that I received the John Marshall email as a result of sharing my email address with you. That said, I'll be the first (or second, in this case) to acknowledge that your sharing my email address with John Marshall was not the only possible explanation for my receiving a email from them, and so those conditions were not sufficient to accuse you.

In fact, I checked my LSAC profile and did in fact authorize them to release my information with member schools, but not non-member schools. This action was an out-of-character oversight; I must have filled out the form hurriedly. Given that circumstance, my accusation was likely unjust and I apologize for jumping to conclusions.

I will adjust my LSAC profile and be more careful in the future before making accusations. Thank you for addressing my concerns so quickly, and, more importantly, for sending the two PrepTests I ordered.

Again, my apologies,

"Jim"


Photo by foolswisdom

Logic and Psychology Today Article about Black Women

LSAT Blog Logic Psychology Today Black Women ArticleA recent article in Psychology Today asked, "Why Are Black Women Less Physically Attractive Than Other Women?" (alt link). It lead to a lot of uproar, which I won't rehash.

However, as I read through the article, I naturally began to construct counterarguments as if the article were composed of many Logical Reasoning stimuli. This is something I often do when I see flawed arguments.

It's also something you should do when reading any article containing arguments - especially when you see an article making strong claims without sufficient evidence and lots of assumptions, as this one does.

More than a few questions came to mind as I read this article.

I'll raise a couple of big ones to get the ball rolling, but I'd really like to see all of you analyze this more in the comments and have some fun identifying the various flaws in the article.

Questions:

The interviewers:

Who were the interviewers rating the various women? To what extent are they representative of the population in general? How many interviewers rated each woman? How many interviewers were there?


The women:

How many women were part of the study? How was the race of each woman determined? To what extent was the sample of women representative of a particular state, region, country, etc.? Were the attractiveness ratings limited to facial features, or do they include body type?


The "explanation" and other unsupported claims:

In the final few paragraphs, the author makes a number of problematic and insufficiently-supported claims regarding BMI, intelligence, genetic mutation, and hormones. He concludes that the supposed racial difference in attractiveness is due to higher testosterone levels in black women because this is the only explanation he can imagine.

In order to improve his argument, he would need to first conduct a perfect (or close to it) study to establish his claim regarding attractiveness, addressing the questions I raised above.

Suppose he was able to do this (no easy feat, given the slippery and normative concepts of both race and attractiveness).

He would then need to systematically dismiss as many other potential explanations for his conclusion as possible. In his article, he only addresses a few, and even those are not supported or sufficiently explained.

***

Your Turn:
What flaws / information gaps do you see in the cited study?

What flaws / assumptions do you see in the author's consideration of (or failure to consider) potential alternative explanations?

What flaws / assumptions do you see in the author's selection of one potential explanation?

What, if anything, did the author do well in making his argument?

What could the author have done better in making his argument?

What could the author have done better with the information at his disposal and/or his topic in general?


Photos by Wikimedia Commons and MiKeARB

LSAT Logical Reasoning Flaw Questions with the Same Argument

Logical Reasoning arguments often contain the same flaw as each other, but such arguments are often about very different topics.

It's somewhat infrequent for different arguments to contain both the same flaw and the very same topic.

In this blog post, I discuss the similarities between two such Logical Reasoning questions from The Next 10 Actual, Official LSAT PrepTests.

Both arguments are associated with Flaw question stems, meaning the question is asking us to identify the flaw, and both are on the topic of altruism and self-interest.

You'll need a copy of the book to follow along as I discuss the following two actual LSAT PrepTest questions:

1. PrepTest 29 (October 1999 LSAT), Section 4, Question 18 (page 41 in Next 10).

Question stem: "Which one of the following most accurately describes an error in the argument's reasoning?"

2. PrepTest 32 (October 2000 LSAT), Section 1, Question 19 (page 124 in Next 10).

Question stem: "A flaw in the argument is that it"

Both arguments reach the same conclusion:

Even behavior that might seem altruistic is actually self-interested.

We see this in the first sentence of the both questions, which contains the conclusion of each argument. The following sentences in each stimulus contain evidence for this.

The PrepTest 29 question suggests people engage in seemingly-altruistic behavior in the hopes of receiving some kind of reward or reciprocal benefit.

The PrepTest 32 question says people engage in seemingly-altruistic behavior in order to boost their self-esteem by feeling useful. Both arguments, on the face of it, seem rather reasonable.

However, the conclusions of both arguments are *too* certain given the way in which the evidence is presented.

The PrepTest 29 question says "can be described" in the 3rd-4th lines.

The PrepTest 32 question says "can be understood" in the 4th-5th lines.

Just because something "can be described" or "can be understood" in a particular way doesn't mean that it must be described or understood in that way. People can interpret actions and behaviors in multiple ways, not only in the ways suggested in these two arguments. The arguments are guilty of the same flaw - they both assume one possible interpretation to be the only possible interpretation, and they fail to consider that there could be other interpretations.


Logical Reasoning: Parallel and Parallel Flaw Questions

Parallel Reasoning questions in the LSAT's Logical Reasoning section don't always mention that there's a flaw in the stimulus.

In other words, not every Parallel Flawed Reasoning question is described as such in the question stem - some simply present themselves as general Parallel Reasoning questions.

However, knowing there is a flaw in the stimulus, even if you have to identify it for yourself, is useful in solving this type of question, as it gets to the core issue of the argument's method of reasoning.

For example, take this LSAT Logical Reasoning question stem and the question itself:

(It's PrepTest 31 (June 2000 LSAT), Section 3, Question 18 - page 100 in Next 10)

"The pattern of reasoning in which one of the following is most similar to that in the argument above?"

There's no mention of anything wrong with the method of reasoning in the stimulus.

If we were to take the claims of some LSAT prep books as true, then we'd falsely assume that the stimulus' method of reasoning is fine, making it harder to solve the question.


However, the question stem of the Logical Reasoning question on page 100 in Next 10 that I just mentioned does not contain any reference to flawed or questionable reasoning. It contains a major flaw.

How could this be?

Well, the LSAT is not obligated to tell you when a stimulus contains flawed reasoning. They often do, but this doesn't mean that they have to.

Whether a method of reasoning is said to be flawed or not, one can still technically find reasoning that is similar to it.

As such, the LSAT creators aren't doing anything wrong by not explicitly stating there's flawed reasoning - they're just making it a bit harder to solve the question by making you recognize the flawed reasoning on your own.


So, how is PrepTest 31 (June 2000 LSAT), Section 3, Question 18 exhibiting flawed reasoning?

To sum it up, the stimulus claims (the following is my interpretation - LSAC does not allow me to publish LSAT questions on the blog itself due to copyright issues):

science requires measuring, and measuring requires units of measurement. Because the unit of measurement that ones uses is arbitrary, therefore, science itself is arbitrary.

However, just because one aspect of science is arbitrary, this doesn't guarantee that science as a whole is arbitrary. It's possible that other aspects of science outweigh the arbitrariness of the unit of measurement selected.

The flaw is like saying that just because carrot cake might include some salt, therefore carrot cake as a whole is salty.

This is often called a part-to-whole flaw. Just because part of something has a certain characteristic, this doesn't mean the entire thing has that particular characteristic.

The correct answer choice exhibits the same flawed method of reasoning as the stimulus - it takes one aspect of a particular pursuit and incorrectly assumes that the pursuit as a whole can then be said to feature that same characteristic.

***

The question from PrepTest 31 that I describe above is not the only case where a Parallel Flawed Reasoning question does not explicitly state in the question stem that the argument contains flawed reasoning.

Another Example: PrepTest 44, Section 4, Question 21

Its question stem states:

"Which one of the following arguments employs reasoning most similar to that employed by the argument above?"

The argument assumes that it is *better* to drive a small car than a large one because smaller ones avoid more accidents in the first place, as they are easier to drive. However, smaller cars don't offer as much physical protection as larger ones, they are easier to drive.

As such, the argument engages in incomplete and questionable reasoning. It fails to consider that perhaps physical protection when one DOES get into an accident is more important than ease of driving. It never explicitly gives a reason why one characteristic should be valued over the other one. The argument would be stronger if it cited as evidence a scientific study or general principle giving support to the value of accident avoidance over protection from accidents that do occur.


Yet Another Example:
PrepTest 48, Section 1, Question 12

Its question stem states:

"The reasoning in the argument above is most closely paralleled by the argument that there is no reason to"

This argument fails to consider that there may still be some legitimate reason or "justification" for delaying the process by which species become extinct even if they will eventually become extinct regardless.

It's like saying that there's no reason to try and keep human beings alive by feeding them or giving them medical care since they will inevitably die regardless, since we're all mortal.

***

The takeaway:

Be aware that even if a Parallel question does not mention that the reasoning contained in the stimulus contains flawed or questionable reasoning, it may do so anyway. Recognizing the flaw in the stimulus is key to recognizing the answer choice containing the reasoning that is most similar.

Negating Conditional Statements in Logical Reasoning

A common technique for solving LSAT Logical Reasoning questions (particularly, Necessary Assumption questions) is to negate each of the answer choices. The correct answer choice, when negated, destroys the argument by preventing the conclusion from logically following from the evidence.

Sometimes, answer choices contain conditional statements, rather than simply containing a single clause.

The proper negation of a conditional statement can often be trickier than the negation of a single clause.

When negating a conditional statement, keep in mind that your goal is NOT to negate the variables themselves.

For example, if we have the statement X → Y, we can do 4 different types of modifications that involve negating the variables included, but none of them is truly a negation of the statement as a whole.

We could negate the sufficient condition, resulting in NOT X -→ Y.

We could negate the necessary condition, resulting in X -→ NOT Y.

We could negate both the sufficient condition and the necessary condition, resulting in NOT X → NOT Y.

We could take the contrapositive of the statement, resulting in NOT Y → NOT X.

However, this isn't what you should be doing when your goal is to negate a conditional statement.

A conditional statement is composed of a sufficient condition and a necessary condition.

It's claiming that one thing is sufficient to guarantee, to require, another thing to occur.

The negation of this concept would be that the thing previously claimed to be sufficient to guarantee another is no longer sufficient.

So, if we had been originally told that X requires Y, the negation of that statement would be that X does not require Y -- that X is no longer sufficient to guarantee Y.

Take the following statement:

If I have pizza, then I will be happy.

P → H

Suppose someone then claims this statement is not true. (For example, they say that if I had pizza, but was repeatedly punched in the face, I wouldn't be happy despite my possession of pizza).

As such, pizza is not truly sufficient to guarantee my happiness, because I also need to not be repeatedly punched in the face in order to be happy.

We can diagram this information as P --/--> H

It's simply P, followed by an arrow with a slash through it, followed by H.

Just as a conditional statement is valid information and useful information, knowing that two particular variables do NOT have a sufficient-necessary relationship is also useful information.

It tells us that one thing alone is NOT enough to require another.

Please see, for example, the diagramming technique used in my recent blog post on Logical Reasoning: Inference Questions and the Contrapositive.

***

Examples of diagramming in this way:

Parallel Flaw question:

Take, for example, a Parallel Flaw question - PrepTest 36 (December 2001), Section 3, Question 19 (page 275 in Next 10).

The evidence tells us that liking turnips is not sufficient to guarantee that one likes potatoes.

We can diagram this as T --/--> P.

(Liking turnips doesn't require that you like potatoes.)

The (flawed) conclusion tells us that liking potatoes is not sufficient to guarantee that one likes turnips.

We can diagram this as P --/--> T.

(Liking potatoes doesn't require that you like turnips.)

Even though this is telling us that we DON'T know something (just because someone likes turnips, this doesn't guarantee that they like potatoes), this doesn't mean that it's not worth writing down or knowing.


Necessary Assumption question:

Additionally, let's look at the choices in a Necessary Assumption question - PrepTest 33 (December 2000), Section 1, Question 19 (page 157 in Next 10). Negating answer choices in Necessary Assumption questions is a useful technique, as I described in my blog post titled, Necessary Assumption Questions, Negation Test, and Must Be True Qs.

Choice A can be rewritten to state, "If a demagogue can enlist the necessary public support to topple an existing regime, then a comprehensive general education system must have been in place" or DNPS → CGES

To negate this, we can say, "a demagogue can enlist the necessary public support to topple an existing regime EVEN IF a comprehensive general education system is not in place" or DNPS ---/--> CGES


Choice B can be rewritten to state, "General awareness of injustice in a society requires literacy" or GAI → L

To negate this, we can say, "general awareness of injustice in society DOES NOT require literacy" or GAI --/--> L


Choice C can be rewritten to state, "If you have a comprehensive system of general education, then you will tend to preserve benign regimes' authority" or CSGE → TPBRA

To negate this, we can say, "Even if you have a comprehensive system of general education, it may not tend to preserve benign regimes' authority" or CSGE --/--> TPBRA


Choice D can be rewritten to state, "If you have a lack of general education, then your ability to differentiate between legitimate and illegitimate calls for reform will be affected" or LGE → ADA

To negate this, we can say, "Even if you lack general education, your ability to differentiate between legitimate and illegitimate calls for reform won't necessarily be affected" or LGE --/--> ADA


Choice E can be rewritten to state, "If a benign regime doesn't provide comprehensive general education, then it'll be toppled by a clever demagogue" or NOT PCGE → TCD

To negate this, we can say, "Even if a benign regime doesn't provide comprehensive general education, it won't necessarily be toppled by a clever demagogue" or NOT PCGE --/--> TCD


***

Please leave your questions for each other about properly negating conditional statements (or negating any kind of statements at all) in the comments and help each other out!


Logical Reasoning: Inference Questions and the Contrapositive

LSAT Blog Logical Reasoning Inference ContrapositiveA while ago, I did a brief post covering one simple format for formal logic inference questions in the LSAT's Logical Reasoning section.

However, they can follow a variety of formats - not simply that particular one.

In this blog post, I'll cover another common type of inference question - one in which the stimulus first sets up a conditional statement. The stimulus then provides us with a clause that activates the sufficient condition of the contrapositive of the previously-provided conditional statement.

***

I know that was a mouthful.

What do I mean?

Suppose a Logical Reasoning stimulus is composed of a conditional statement, like X -> Y, as well as the additional information, NOT Y.

We can then take the contrapositive of that conditional statement, NOT Y -> NOT X, and plug in the additional information of NOT Y to spit out the result, NOT X.

***

Must Be True Example

This is all a bit abstract, but let's look at it with an example from a real LSAT Logical Reasoning question, a Must Be True question:

(Please see PrepTest 30 (December 1999 LSAT), Section 2, Question 20 - page 59 in Next 10 to follow along. Due to copyright law, I'm unable to reproduce the text of the question itself.)

This stimulus begins with a claim made by critics saying (paraphrased):

If continued public funding is justified, then there must be some indication of public benefit.

We can diagram their claim with the following symbols:
CPFJ -> IPB
The stimulus continues by saying that if the critics' claim is true, then there would not be tremendous public support.

We can diagram this statement with the following symbols:
"CPFJ -> IPB" -> NOT TPS
Rather than using the word "NOT", I'd probably draw it instead like this:

LSAT Blog Logical Reasoning Inference Contrapositive


I placed the critics' claim in quotes to indicate that the truth of their conditional statement's claim is serving as the sufficient condition of another conditional statement - the one with the necessary condition being (paraphrased), "then there would not be tremendous public support."

However, it doesn't simply say then there wouldn't be tremendous public support - it says, there wouldn't be all the tremendous public support that we do, in fact, have.

In other words, we DO have tremendous public support (diagrammed as):
TPS
As such, we can now take the contrapositive of the big conditional statement we just talked about:
"CPFJ -> IPB" -> NOT TPS
If there is tremendous public support, then the critics' claim that "in order for continued public funding to be justified, we must have an indication of public benefit" is NOT correct.

In other words, if we have tremendous public support, then an indication of public benefit is not necessary in order for continued public funding to be justified.

We can diagram this as:

TPS -> NOT "CPFJ -> IPB"

Again, rather than using the word "NOT", I'd probably draw it instead like this:

LSAT Blog Logical Reasoning Inference Contrapositive


Since we do have tremendous public support, then we can conclude that the critics 'claim is not correct.

NOT "CPFJ -> IPB"

Choice E of this question pretty much says just that.

***

Recommended assignment:

Now go through this process with another Logical Reasoning question that follows a similar format. I recommend PrepTest 31 (June 2000 LSAT), Section 3, Question 22 - page 101 in Next 10.

Arguments and Contrapositives | Necessary and Sufficient Assumptions

LSAT Blog Arguments Contrapositives Assumptions Necessary SufficientI spend a great deal of time talking about the difference between Necessary Assumption and Sufficient Assumption questions in the LSAT's Logical Reasoning section.

Arguments assume a link between the evidence and conclusion presented - this link can often easily be framed as a conditional statement.

Because the contrapositive of this statement is simply a rewording of the argument itself, the contrapositive of that statement is both necessary and sufficient for that argument to work.

As such, it can serve as the correct answer to both Necessary Assumption and Sufficient Assumption questions.

***

Necessary Assumption
Let's start with the fact that the contrapositive of an argument's evidence-conclusion link can serve as a necessary assumption.

I mean that if X -> Y is an argument, then NOT Y -> NOT X is a necessary assumption (an assumption required) for that argument to be valid.

After all, if the contrapositive were negated, then the original statement would not be valid either, and the argument wouldn't be valid. As such, the original statement requires the contrapositive to be true as well.

***

Sufficient Assumption

Additionally, the contrapositive of an argument's evidence-conclusion link can serve as a sufficient assumption.

I mean that if X -> Y is an argument, then NOT Y -> NOT X is an assumption that is sufficient for the argument to be valid. What I mean is that if we're told, as new information, that NOT Y -> NOT X is valid, it must be the case that the argument itself (X -> Y) is also valid. This is because if the contrapositive of a statement is valid, then the original must also be valid, since they're logically equivalent.

This is all a bit abstract, but let's look at it with a couple of examples from real LSAT questions:

Necessary Assumption example:

(Please see PrepTest 36 (December 2001 LSAT), Section 3, Question 16 - page 275 in Next 10)

In this argument, the stimulus tells us (paraphrased):

Because reptiles can't make big behavioral changes when the environmental changes a lot, reptiles aren't capable of engaging in advanced thought

In shorthand, the argument is saying:

NOT capable of big behavior changes with environmental changes -> Not capable of complex thought

The contrapositive of this statement would be something like:

Capable of complex thought -> capable of big behavioral changes with environmental changes

In other words:

If an animal is capable of complex thought, then it must be capable of making big behavioral changes as the environment goes through big changes.

Choice D of this question pretty much says just that.

Again, if an original conditional statement that forms the core of an argument is considered to be true, then it is required that its contrapositive also be true in order for that argument to work.



Sufficient Assumption example:

(Please see PrepTest 36 (December 2001 LSAT), Section 1, Question 26 - page 261 in Next 10)

In this argument, the stimulus tells us (paraphrased):

Because Vermeer used expensive props, it must not be due to a scarcity of props that he kept using the same props over and over.

In shorthand, the argument is saying:

$ props -> NOT due to small # of props that V kept reusing them

The contrapositive of this statement would be something like:

If it were due to a small # of props that V kept reusing them, then NOT $ props.

In other words:

If it were due to a small number of props that Vermeer kept reusing the same ones, then he wouldn't have been using expensive props in the first place.

Choice E of this question pretty much says just that.

Again, if we're told, as new information in an answer choice, that the contrapositive of the argument is guaranteed to be true (or is "assumed"), then the original version of that conditional statement (the one in the argument) must also be true, and the argument is valid.

Photo by mitopencourseware

Logical Reasoning: Necessary and Sufficient Conditions

LSAT Blog Logical Reasoning Necessary Sufficient ConditionsLSAT Blog reader Vicky recently wrote with the following:

"I know you have already covered the sufficient and necessary conditions quite extensively, but there are still many (myself included) who are still stymied by it.

If you feel that the explanation I provide below is accurate and acceptable, posting it as an introductory Logical Reasoning topic might help other LSAT students."

I found Vicky's explanation to be useful and have included an edited version of it below.

Please thank Vicky in the comments for sharing her tips with everyone!

Vicky's Tips on Understanding Sufficient / Necessary Conditions:

Dealing with the concepts of Sufficient and Necessary conditions becomes a whole lot easier if I think of them in mutually-exclusive terms.

I consider the introduction of a new entity, the outcome/objective of these conditions, and, in some instances, with the sufficient condition being a specific example/small part of the bigger necessary condition.

For example, the limo example from Steve's blog post titled, LSAT Logic: Sufficient vs. Necessary Conditions: If I take a limo somewhere, I'll go somewhere

My interpretation is this:

- go somewhere = outcome/objective

- take a limo somewhere = sufficient condition

- traveling somehow, by any means possible, (such as: walk, limo, spaceship, teleport, have a friend wrap you up in a bouncy material and roll you across town to your destination...) = necessary condition

- taking a limo somewhere is a form of traveling. However, taking a limo somewhere is not the only form of traveling.

As such, the sufficient condition is a specific example/subset of the necessary condition. Traveling in general doesn't require that you take a limo somewhere. Like Darth Vader and Star Wars. Like Superman and superheroes. Like LSAT and traditional law school admission. (Do I need to seek professional help here?)


From knowing you got somewhere, we know/safely assume the necessary condition is met - you traveled. However, we can't automatically assume that you took a limo somewhere

From knowing that you took a limo somewhere, we can safely assume you got somewhere - the outcome is met. This also automatically satisfies the necessary condition of traveling.

The necessary condition is automatically met when the sufficient condition is met.

If the necessary condition is not met, the sufficient condition is automatically not met.

The necessary condition might be met without the sufficient condition being met.

This is relevant to page 25 of the LSAT SuperPrep / pages 30-31 of the LSAT Handbook. On these pages, LSAC says there are only 2 possible inferences that can be made from a statement that includes a necessary condition and 2 from a statement that includes a sufficient condition.

NC = necessary condition
SC = sufficient condition

NC #1. Infer from knowing that the NC is NOT met, the outcome does NOT occur = Not NC --> not outcome

NC #2. Infer from knowing that if the outcome occurs, the NC is met = outcome --> NC

SC #1. Infer from knowing that if SC is met, the outcome occurs = SC --> outcome

SC #2. Infer from knowing that if the outcome does not occur, the SC is not met = Not outcome --> not SC

***

“If I take a limo somewhere, I'll go somewhere.”

Scenario 1: I did not travel. So I didn’t go somewhere. I obviously didn’t take a limo somewhere

Not NC --> Not outcome --> Not SC

Scenario 2: I took a limo somewhere. So I went somewhere. I've obviously traveled.

SC --> outcome --> NC

Photo by livenature

Complete Logical Reasoning Explanations Update

I've added explanations for the Logical Reasoning sections of LSAT PrepTests 61 and 62 (the October 2010 LSAT and December 2010 LSAT) to that mega-PDF - the Complete Logical Reasoning Explanations for Recent PrepTests.

If you haven't bought them yet, be sure to check out more about these Logical Reasoning explanations.

LSAT Numbers: All, Most, Several, Many, Some, None

LSAT Blog Numbers All Most Several Many Some NoneIsn't it annoying when words seem to mean something different on the LSAT than they do in real life?

Starts to make you wonder about the last time an LSAT test-writer spoke with a live human being.

In this post, I clear up some of the differences between our normal understandings of common quantifiers (words that indicate the number of something) and the way the LSAT uses them.

***

All

The word "all" isn't one of the confusion-causing words, but let's cover it anyway.

Let's suppose I've got 100 chocolate-chip cookies in a box, and, by the time I finish writing this blog post, I've eaten every single one of them (writing works up an appetite, don't judge). Then, I can say with certainty, "ALL the cookies in that box were deliciously fattening."

All = 100%


Most / Majority

Let's suppose I've exercised a bit of restraint and only eaten 99 of them (I'll eat the remaining one after the February 2011 LSAT.) I can then say with certainty, "Most of the cookies in that box were finger-lickin good." I can say "most" because I've eaten a majority of them. However, until I eat the remaining one, I won't be able to tell you whether all of them were good or not, because I haven't thoroughly, ummm, "examined" each one.

As such, it's entirely possible that all of them are good, so when I say that most of them are good, we still have to allow for the possibly that all of them will be good. This is why the word "most" allows for the possibility of all.

(The same would be true if I ate 50 cookies and then took just a tiny nibble of the 51st, because I'd then be over the halfway point. At that point, I can say that a majority are good, but it's still possible that all are.)

In everyday speech, when we say things like "most of that movie was pretty good" and "most of that meal was delicious" there's an implicit (assumed) meaning that not all of it was good.

If we wanted to speak literally all of the time, we'd say things like, "most, but not all, of that movie was good. I found the ending rather elementary, old chap" or "while the majority of my dinner was delectable, the crème brûlée was a bit overdone."

However, we don't always elaborate at the outset, because then I'd have to punch you in the face for speaking like Sherlock Holmes and complaining about your fancy crème brûlée. Instead, for purposes of simplicity, we usually just emphasize the words "most" and "majority," and the other person usually asks us which parts we didn't like.

If we wanted to take those everyday sentences, with their everyday meanings, but give them just a small dose of literalism, we'd say, "most, but not all, of that movie was pretty good" and "most, but not all, of that meal was delicious."

Without the "but not all", when I hear you say, "most of that movie was pretty good", it's possible that you're simply just-over-halfway through the movie and think everything so far is good

Most / Majority = A range from 1/2 of total + 1 (or 1/2 plus the smallest possible unit that can be broken off, like a cookie crumb) - 100%

For purposes of simplicity, we might just think of it as 51% - 100%.



Several / Many

If I told you that I have a box of a 100 chocolate-chip cookies, I confirmed that several of them are tasty, you wouldn't truly know how many I ate, or how many of them are actually tasty.

"Several" and "many" refer to some kind of sizable (and plural) number, so we know it's more than one or two, but how many exactly? It's impossible to say. This is an indeterminate number. Like most/majority, it allows the possibility of all.

Several / Many = a range of more than 2 - all the way up to 100%

For purposes of simplicity, we can think of it as 3 - 100% or 3 - all.



Some

Let's suppose I catch you stuffing your face with cookies from that 100-cookie box. I ask, "How many did you eat?" You reply, ""Some..."

Vague, right? Maybe you ate only 1, or maybe you had 5, 10, 49, 75, 99, or 100. Without more information, we don't know just how many you ate.

Like the many/majority example, making a claim regarding "some" does not exclude the possibility that "all" have that characteristic, whether it's with regard to how many of them were delicious or just how many were eaten.

In order to know that you hadn't eaten all the cookies, you would've needed to specifically claim that you had eaten "some, but not all", so I'll know that there's still at least 1 cookie remaining for me to eat.


Some = a range from 1 - all the way up to 100%

For purposes of simplicity, we can think of it as 1 - 100% or 1 - all.



None

The word "none" isn't one of the confusion-causing words, but let's cover it anyway.

Let's suppose I've got a new box with 100 chocolate-chip cookies, but I now have a stomachache from eating all the cookies in the previous examples. I can't even bear to look at this new box of cookies without thinking about how I'll soon be another number in the oft-cited statistics about America's obesity epidemic.

So, I take the box of cookies and donate it to the homeless guy on the street corner (a questionable donation, I know, but I didn't think he'd want kale).

How many cookies did I eat from that box? None. Zero. Zilch. Nada. How many of the cookies in that box can I say are delicious with absolute certainty? I don't know. Maybe they're stale, and the homeless guy will get pissed at me.

None = 0%

***

Photo by Lisa W.

LSAT PrepTest Question Explanations for Logical Reasoning (a free sample!)


I've written explanations for over 1,000 LSAT questions. Below, I'm including a small free sample of the Logical Reasoning explanations just so you can see what they're like.

Get the full LSAT PrepTest explanations for LSAT PrepTest 65 (and TONS of other exams) HERE.

These are just for the first Logical Reasoning section of LSAT PrepTest 65:


Section 1

1. Question Type: Strengthen

Gap: The argument concludes that a diet doesn't necessarily have to be extremely low in fat in order to protect the heart. This is based upon evidence that those following the Mediterranean diet were less likely to have a 2nd heart attack than were those in the other group, even though the Mediterranean diet contains fat. In order to strengthen this argument, we're looking for new information to help connect the Mediterranean diet (and its fat) to a lower risk of heart attack.

(A) This just promotes another route one could take in attempting to reduce the risk of a second heart attack. It certainly doesn't do anything to promote the idea that the Mediterranean diet can aid in the health of one's heart.

(B) See above. This tells us that the particular oils in the fat found in the Mediterranean diet actually *aids* the health of the heart, rather than harming it. If this is true, then a diet certainly doesn't need to be low-fat in order to protect the heart.

(C) We're not concerned with whether the study's participants continued following the Mediterranean diet after the study concluded. We're concerned with the implications of the study's results on the relationship between fat consumption and heart attack risk.

(D) We're not concerned with other factors (such as exercise) in addition to diet. We're specifically concerned with the relationship between diet and heart health.

(E) Whether drugs can help in addition to the Mediterranean diet is irrelevant. We're only concerned with the Mediterranean diet and the fats/oils associated with it.

Correct Answer: (B)


2. Question Type: Conclusion

Among other things, this stimulus contains both evidence (1st and 2nd sentences of stimulus) and an intermediate conclusion (3rd sentence of stimulus). The intermediate conclusion is supported by the 2nd evidence sentence, and it goes on to support the main conclusion itself, which is the first half of the stimulus' final sentence. The second half of that final sentence also serves as evidence for the main conclusion, which immediately precedes it.

(A) See above. This statement is directly support by the immediately-preceding sentence, which, in turn, is supported by the stimulus' 2nd sentence. The entire stimulus is concerned with promoting the idea that florists should plan ahead and stock up on white carnations.

(B) This is the second sentence of the stimulus, which simply serves as some basic evidence - there aren't any statements that support this one.

(C) This first sentence of the stimulus simply serves as some evidence. There are no statements that support it, so it can't be the overall conclusion.

(D) This serves as evidence for the main conclusion that florists should stock up on the white carnations.

(E) This is our intermediate conclusion. While it's supported by the second sentence, it also serves as support for the following sentence. Since it serves as evidence for something else, it can't be the main conclusion itself.

Correct Answer: (A)


3. Question Type: Evaluate the Argument (EXCEPT)

The wording of this question stem is a bit unusual. For this question, we're really just looking for the four choices that could help us evaluate the argument. Whatever remains is our answer, as it doesn't really help us to evaluate the argument (and would be the choice least useful in helping us to evaluate it).

This argument essentially claims that everyone should use low-wattage bulbs even though they cost more because they have a variety of enormous advantages for the homeowner. In order to evaluate the argument, we'd want to know, among other things, what those advantages are, as well as how they relate to the money one has to invest in the bulbs.

(A) The cost of electricity associated with using each bulb is certainly relevant to determining which type of bulb is truly better for the homeowner.

(B) The profits the industry folks expect to make from sales of these bulbs are not relevant to whether everyone should actually use them or not. It's tempting to eliminate this choice as one that *is* relevant because one might think "well, they're obviously biased in advocating for it (in the middle of the stimulus)." However, just because they'd happen to benefit from these sales, it doesn't have anything to do with whether low-wattage bulbs are actually the better option. To say otherwise would be to commit an ad hominem attack (attacking the personal characteristics of the source of an argument, rather than considering the argument on its own merits).

(C) Knowing how much more the low-wattage bulbs cost as compared to the regular ones is important to determining which is better. If the low-wattage bulbs were significantly more expensive, that would certainly weaken the argument, and if they were only a little more expensive that would strengthen the argument.

(D) The experiences of those who've already experienced the difference would certainly be relevant as well. Perhaps the light that low-wattage bulbs give off is low-quality in some way that detracts from the experience of those who use them.

(E) Learning how the life of each bulb is very important to determining whether a homeowner should use the low-wattage bulbs. If the low-wattage bulbs burn out very quickly, then the homeowner would have to keep buying new ones.

Correct Answer: (B)


4. Question Type: Conclusion

The stimulus features an argument by analogy. It's giving us evidence in the form of the swimming pool example to give us a general principle that it claims should be applied to the soft drink/candy example. It argues that it's more important to teach children what to do instead of forcing them to do what's appropriate. As such, the stimulus is arguing in favor of *teaching* children why they shouldn't consume soft drinks and candy instead of solely restricting the products.

(A) Nothing in the stimulus supports the idea of TV being a good source of information. TV is simply mentioned in passing as a source of advertisements of the soft drinks and candy.

(B) There's no evidence in the stimulus to suggest this. Again, TV is mentioned in passing as a source of advertisements of the soft drinks and candy.

(C) See above. The stimulus is concerned with demonstrating that it's more important to teach children the nutrition relevant to consumption of soft drinks and candy than it is to simply restrict consumption.

(D) The stimulus doesn't suggest anything about physical activity - the principle we're to extract from the swimming example doesn't suggest that the children in this example should be taught behaviors such as these.

(E) The principle we're to extract from the swimming example doesn't suggest avoiding the source of the problem altogether. Rather, it suggests learning how to deal with the issues it raises.

Correct Answer: (C)


5. Question Type: Weaken

Gap: The argument assumes that simply because 2/3 of those interviewed by TV program were against the proposed freeway, that the program itself is biased against the freeway. It's failing to consider that perhaps this is a fairly accurate representation of the general opinion of those affected by the proposed freeway. Or perhaps maybe even more than 2/3 of those affected by the freeway are against it, so the fact that 2/3 of those interviewed were against it was actually under-representing their numbers.

(A) Whether those watching the program were aware beforehand or not is irrelevant. We're concerned with whether the percentage of those depicted on the program against the freeway is indicative of bias on the part of the TV program.

(B) The viewers' expectations are completely irrelevant to potential bias on the part of the TV program.

(C) We don't know whether their emotion would indicative of greater or lesser bias on the part of the TV program, so this can't be said to affect the argument.

(D) See above. This suggests that the actual percentage of those against the proposed freeway is somewhat in line with general opinion. If a significant majority of the public were in favor of it, then the stimulus' argument would have a better case for accusing the TV program of being biased. However, learning that this is *not* the case weakens the argument significantly.

(E) This strengthens the conclusion (even if only a little bit), rather than weakening it, by suggesting that the TV station is *more* likely to be biased, rather than against it. In any event, this still doesn't involve the evidence about the percentage of those interviewed who were against the proposed freeway.

Correct Answer: (D)


6. Question Type: Parallel Reasoning

The stimulus gives us a situation where someone is unsure of whether something is true, but the individual errs on the side of caution by extending the benefit of the doubt to another.

(A) This choice exhibits a line of reasoning similar to that described above, making it parallel to the stimulus. The individual isn't sure whether he or she repaid Farah, but this individual errs on the side of caution and decides to repay Farah just to be safe.

(B) This choice is simply concerned with the speaker's expectations - not with erring on the side of caution to *perform* a particular action.

(C) The speaker gives a reason why he or she opposes a particular course of action, but there's no admitted ambiguity similar to that of the stimulus.

(D) This choice features a line of reasoning arguing in favor of a particular course of action and even explains how it could be financed, but, again, there's no admitted ambiguity or erring on the side of caution similar to that in the stimulus.

(E) This choice gives us an example of a person whose writing was not adversely affected by her personal experiences in a particular way, but none of this has anything to do with erring on the safe side and extending the benefit of the doubt to another.

Correct Answer: (A)


7. Question Type: Conclusion

The stimulus features an argument by analogy. The evidence in the form of the medicine example gives us a general principle that it claims should be applied to the topic of government intervention in the free market. We can extract the principle from the medicine example that a particular course of action has both positive and negative effects, the course of action should only be pursued if the positives significantly outweigh the negatives. As such, we can infer that the argument is concerned with demonstrating (in its conclusion) that government intervention in the free market should only be pursued if the positives of doing so significantly outweigh the negatives.

(A) The popularity of a course of action among those affected is not a principle covered by the stimulus.

(B) The medicine example doesn't set such a high standard for taking a course of action. Rather, it lays out the positives significantly outweighing the negatives as the requirement for pursuing a course of action.

(C) It might be perfectly fine for the course of action (government intervention in the free market) to significantly worsen existing problems if the positive consequences of such intervention significantly outweighed those negative consequences.

(D) See above.

(E) The concepts of solutions and unsolvable problems are never discussed by the stimulus.

Correct Answer: (D)


8. Question Type: Flaw

Flaw: This argument is guilty of a correlation-causation flaw. It assumes that just because there's a relationship between a relatively lower consumption of fat calories and those who read nutrition labels, that the label-reading is what's *causing* people to consume fewer fat calories. It's entirely possible that both of these qualities are simply due to the fact that these people are health-conscious.

(A) See above.

(B) The argument doesn't make use of any kind of sample. This stimulus is not describing a study. Rather, it's simply talking about this category of people in general.

(C) This describes the flaw of confusing necessary and sufficient conditions, but this argument doesn't feature conditional statements, so it can't be guilty of this flaw, as it doesn't feature any necessary or sufficient conditions at all.

(D) The argument only features one explanation (a causal relationship) - not two.

(E) The argument never discusses anyone's intentions.

Correct Answer: (A)


9. Question Type: Conclusion

The argument concludes that it's unlikely that Apatosaurus was able to gallop. It uses the fact that Apatosaurus' leg bones wouldn't have been able to deal with stress of galloping, and would've broken as a result, as evidence, and this evidence is supported by experiments with modern bones that were similar in key respects to those of Apatosaurus.

(A) This is tempting, as it is an intermediate conclusion, supported by the experiments. However, this is used to support the main conclusion, that it's unlikely that Apatosaurus would've been able to gallop.

(B) This evidence serves to support the claim at the beginning of the stimulus' 2nd sentence.

(C) See above.

(D) This serves as a general summary of the argument (when combining the intermediate conclusion and main conclusion). As such, this choice is going beyond just the main conclusion itself, which is simply the final clause of this choice.

(E) This is a fact would strengthen the validity of the experiments conducted with modern bones, but there's no evidence supporting this statement in the stimulus, so it can't be the main conclusion.

Correct Answer: (C)


10. Question Type: Role of the Statement

The stimulus claims in its second sentence that the advance of fortifying the salt with iron could reduce anemia caused by iron deficiency. The stimulus then goes on to provide some facts about salt that support this claim. The statement in question is one of these facts, so it serves as evidence for the argument's conclusion.

(A) The statement in question supports the argument's conclusion about the advance helping to reduce anemia due to iron deficiency. As such, it's not the conclusion itself.

(B) See above.

(C) The argument never attempts to disprove this statement. Rather, this statement supports this argument's conclusion, so it is part of the argument.

(D) This statement doesn't limit the argument's conclusion in any way - it supports the argument's conclusion, giving us reason to believe it's true.

(E) It doesn't serve as any kind of example. It's a broad statement that gives us reason to believe that salt fortified with iron could be a viable way to reduce anemia caused by iron deficiency.

Correct Answer: (B)


11. Question Type: Parallel Flaw

For this question, we must find the flaw in the stimulus and then look for the answer choice exhibiting the same flaw.

Flaw: In the stimulus, the inspector fails to consider an obvious possibility - perhaps Mr. Tannisch stole his guest's diamonds himself. As such, we're looking for an answer choice where there's an obvious potential culprit that the argument fails to consider.

(A) See above. In this choice, the argument fails to consider the possibility that perhaps the food from the camp cafeteria is the cause of the campers' illness.

(B) This argument actually doesn't exhibit any kind of flaw. It's perfectly reasonable, and its conclusion is moderate ("might have").

(C) This argument just takes past trends as being somewhat indicative of future ones. The conclusion is a little strong ("unlikely") given the simple fact that there are multiple swimmers, it's at least somewhat likely that one of them might win. In any case, this isn't the same flaw as that of the stimulus.

(D) This argument fails to consider that the cause of the cavities on the left side of her mouth may be due to causes other than chewing on that side. It's simply assuming that chewing must have been the cause. However, the stimulus doesn't assume that one specific factor must be the cause to the exclusion of others. Rather, it fails to consider one obvious cause.

(E) This argument is flawed because it assumes without providing adequate evidence that the peas will probably perform similarly to the tomatoes. However, we don't have any kind of comparison without warrant in the stimulus, so this flaw isn't similar.

Correct Answer: (A)


12. Question Type: Resolve the Paradox

This question is asking us to help explain why different shrimp populations at the different reefs have significant genetic differences, given that we would've expected the currents to mix the populations, leading them to be genetically similar. A choice that explains how mixing (due to the currents) could occur, yet interbreeding still *not* occur between the different populations, would help to resolve the paradox.

(A) Even if this were true, it still wouldn't do anything to help explain why the shrimp of different populations in these reefs aren't interbreeding but are instead remaining separate, developing genetic differences. The information provided by this choice is both unsurprising and unhelpful. It'd likely be the case with regard to any species that animals within one species differ less genetically than do animals of different species. 

(B) We're not at all concerned with what happens within a shrimp population. We're only concerned with what's going on *between* the different shrimp populations at these 11 different reefs. Also, note that the choice says "differ genetically" - all it's telling us is that these shrimp aren't genetically *identical* (like clones or twins). As with (A), this isn't telling us anything we wouldn't already reasonably assume based on a common understanding of genetics one would learn in a middle school biology class.

(C) See above. If we learned that the shrimp return back to their own reefs for breeding, this would prevent the interbreeding expected in the stimulus, allowing the genetics of each population to diverge.

(D) Learning that shrimp may leave their reef of birth upon reaching maturity doesn't tell us anything about why the different populations aren't mixing genetically.

(E) This choice is somewhat similar to (D). Learning that some shrimp leave their reef of birth and don't remain within one of the 11 different reefs still doesn't explain why those who *do* remain aren't mixing with shrimp from other populations.

Correct Answer: (C)


13. Question Type: Strengthen

Gap: The argument concludes that seawater farming near the sea level should be cost-effective in desert regions based upon the evidence that pumping seawater into such farms is relatively cheap. It would help us to strengthen this argument if we learned that pumping seawater into such farms is a significant cost associated with farming. This new information, combined with the fact that we already know pumping seawater into such farms is relatively cheap, would increase the likelihood that these farms are cost-effective.

(A) We're not concerned with the relative nutritional value of different types of plants. Rather, we're concerned with strengthening the conclusion related to the cost-effectiveness of certain types of crops.

(B) The fact that salt is necessary as opposed to tolerable for halophytes is irrelevant - we're concerned with the costs associated with seawater agriculture near sea level in desert regions.

(C) These research expenditures wouldn't necessarily be related to the costs associated with conducting the seawater agriculture described - and if, for some reason, they were, that would likely weaken the argument regarding cost-effectiveness, rather than strengthening it.

(D) Variation in the costs associated with growing halophytes in different types of farms is unimportant, as we don't know *how* the costs differ. Are they cheaper to grow by means of seawater irrigation? Are they more expensive to grow in this way? The answer choice doesn't tell us, so, by itself, it doesn't affect the argument either way.

(E) See above. Learning that such pumping of water is a proportionally-large cost associated with farming certainly increases the likelihood that seawater farming near sea level in desert regions can be cost-effective, given that pumping water in this type of area is cheaper than in other types of regions.

Correct Answer: (E)


14. Question Type: Strengthen

The general principle in the stimulus tells us that if a contract is written in such a way that a reasonable customer wouldn't bother to read it thoroughly before agreeing to it, then the law should privilege what one would've reasonably expected it to say, rather than what it actually *does* say. The application suggests that the insurance company should be required to pay for damage to Celia's car caused by hail, even though the contract specifically excluded hail damage.

The missing piece here that would make the principle relevant to the application, strengthening it, would be if we learned that the contract that Celia signed was one that a reasonable person wouldn't have bothered to read thoroughly. If we learned that this was the case, then it allows us to make sense of the statement that the insurance company should have to cover the hail damage to Celia's car, even though the contract excluded it (because we're now privileging what the policyholder would've expected, rather than what the policy actually stated.

(A) and (D) These doesn't explicitly tell us that the contract was written in such a way that a reasonable person wouldn't read it thoroughly before signing it.

(B) See above.

(C) and (E) This doesn't tell us that it would be reasonable for someone to expect hail damage to be covered by the policy. (With regard to (C), the fact that Celia *did* read the contract is irrelevant.)

Correct Answer: (B)


15. Question Type: Flaw

Flaw: The argument assumes that simply because iatrogenic disease accounts for half of all deaths, that if we were to prevent all iatrogenic disease, the number of deaths per year would decrease by 50%. It's failing to consider that at least some of the people who would've died from iatrogenic disease will instead die from something else (perhaps whatever it was that they were in the hospital for in the first place). It's not as if all of these people would suddenly be immortal if they were prevented from dying due to iatrogenic disease.

(A) This category of disease is completely irrelevant. The stimulus isn't concerned with preventing noniatrogenic disease. Even if it did consider the effect of the prevention of noniatrogenic disease, the fact remains that the stimulus is failing to consider that which is described above.

(B) Even if some can be replaced by less risky alternatives, this still doesn't address the argument's assumption about cutting the number of deaths per year in half.

(C) See above. Even if they're won't die from iatrogenic disease if it's to be cured, this doesn't mean that they won't die from something else.

(D) The stimulus accounts for this by introducing the idea with "if". Whether it's possible to do so or not is irrelevant.

(E) Whether iatrogenic diseases are as common as this choice implies is irrelevant. We're concerned with addressing the argument's conclusion about cutting the number of deaths per year in half.

Correct Answer: (C)


16. Question Type: Sufficient Assumption

Evidence: 

(If we had the statement "A or B", we could diagram it as:

NOT A -> B
NOT B -> A)

As such, the first sentence of the stimulus can be diagrammed as following:

NOT Vote Against -> Abstain
NOT Abstain -> Vote Against

The second statement tells us:

All Abstain -> Decided by Voters

Combining this with the first conditional above, we get:

NOT Vote -> Abstain
If ALL Abstain -> Decided by Voters

Faced with these conditionals, the activist concludes that at least one city councilmember should vote against.

Learning that the proposal shouldn't be decided by voters would explain (and fully justify) the activist's conclusion that at least one city councilmember should vote against.

As such, the principle that the proposal shouldn't be decided by voters is the ideal sufficient assumption, and it's what we're looking for in the answer choices.

(A) This is referring to the second sentence of the stimulus, as well as telling us which way the city's voters would vote. However, this has nothing to do with the activist's recommendation in the conclusion regarding what at least one city councilmember should do. And we're not concerned with what the voters would do. The stimulus is primarily concerned with the city councilmembers.

(B) See above.

(C) The stimulus is not concerned with what the city councilmembers *will* do. Rather, it's concerned with what they *should* do.

(D) This is simply a negation of the stimulus' 2nd sentence, and it still doesn't have anything to do with the activist's recommendation in the conclusion regarding what at least one city councilmember should do.

(E) This doesn't really fit any of the parts of the argument. The conclusion tells us that *at least* one member should vote against - it's not concerned with what should happen if *exactly* one votes against. Additionally, this doesn't relate the conclusion back to the evidence, so it's not doing anything to guarantee the conclusion's validity.

Correct Answer: (B)


17. Question Type: Flaw

Flaw: The argument fails to consider that perhaps people's confidence in their own personal financial situations is directly related to their confidence in the overall economy. If this is the case, then the media critics aren't necessarily mistaken.

(A) This choice has it backwards - the argument should be considering how people's perception of the overall economy affects their views of their own situations, not vice-versa.

(B) The argument never made any assumption or claim regarding the extent to which such news reports are accurate. The argument's more concerned about whether the reports impact people's views of their personal financial situations.

(C) This is along the lines of negation and distortion of different pieces of information in the stimulus advanced by the critics. They tell us that people's views of their own economic situation can be affected by hearing of information from economic reports, but this doesn't mean that anything close to the opposite of that needs to be considered. The argument isn't concerned with those who *don't* pay attention to economic reports - we're only concerned with those who do.

(D) See above. This suggests that if people take a negative view of the overall economy, then they'll also take a negative view of their personal finances. As such, we can say that their opinion of the former is closely related to their opinion of the latter.

(E) We're not concerned with whether people are affected by economic downturns differently depending on their degree of awareness of what's coming - this isn't specifically related to the critics' claim that news reports can affect people's perceptions of the economy.

Correct Answer: (D)


18. Question Type: Necessary Assumption

Gap: The argument assumes that because people must have already tried to domesticate every large mammal species currently in existence, most of those around today would probably be hard to domesticate or not worth the trouble. It fails to consider that perhaps human understanding of domestication has significantly advanced in the past few thousand years. If this were the case, then perhaps domesticating these animals would be worth another shot, since it might be much easier to domesticate them with modern technology and techniques.

(A) We don't need them to have tried to domesticate every *single* wild large mammal species. The conclusion is limited to *most* wild large mammal species.

(B) See above. In order for the conclusion to be valid that domesticating these animals is too hard or not worth doing, it must be the case that it's not much easier to domesticate these animals now then it was back then.

(C) The stimulus doesn't *require* that some of them went extinct. Even if none of them had gone extinct, that wouldn't affect the argument in the slightest. Either way, it's only concerned with those still in existence today.

(D) The argument doesn't impose any relationship *between* the ease of domestication and the benefit derived from doing so - it simply mentions both of these factors in its conclusion.

(E) While the stimulus implies that the ones that are easier to domesticate have probably been domesticated already, it doesn't require that they were domesticated in order of ease of domestication.

Correct Answer: (B)


19. Question Type: Strengthen

Gap: The argument assumes that this year's bird population must be larger than usual *because* of the mild winter. The mild winter allowed birds to stay in the region instead of migrating, *and* it allowed them to forage instead of using the bird feeder. Something relating at least one of these facts about the mild winter to a larger-than-usual bird population would help to strengthen the argument.

(A) This doesn't tell us *which* specific weather patterns lead to increases in bird populations.

(B) This doesn't tell us *how* the mating behaviors differ. If this choice had told us that when birds don't migrate south, more mating occurs in general (leading to population increase), *that* would strengthen the argument.

(C) See above. This relates foraging to survival because it's telling us that using the bird feeders (as they would've done in a harsher winter) would make them more vulnerable to predators, leading to a relatively lower population.

(D) This actually weakens the argument, rather than strengthening it, by suggesting that birds that don't migrate south, but instead remain where they are would have less food available to them during the winter. This would likely lead to a population *decrease* rather than a population increase.

(E) This has no impact on the argument. We're not concerned with where they get their food from unless we we're given specific reason to believe one source is better than another.

Correct Answer: (C)


20. Question Type: Flaw

Flaw: The argument fails to consider the possibility that newspapers cover small observational studies more frequently than larger randomized trials not because the small observational studies have a higher likelihood of dramatic findings than do large randomized trials, but because there are simply many more small observational studies, period. It's entirely possible that small observational studies are just as likely (or even less likely ) to have dramatic findings as are large randomized trials, but it's simply due to the much-larger *numbers* of small observational studies that there's a larger number of such studies with dramatic findings.

(A) The argument doesn't question the motives of those reporting study findings (it never even mentions these individuals, who would likely be scientists).

(B) The journalist's doesn't question the validity of any study's findings merely based upon how dramatic the results might seem.

(C) The argument isn't guilty of any such confusion - it simply states that there's a greater absolute number of newspaper stories about small observational studies than there are stories about large randomized trials *because* small observational studies are more likely to have dramatic findings.

(D) See above.

(E) The journalist never discusses *why* results might sound dramatic - it simply states that certain studies’ findings do sound dramatic (1st sentence of stimulus), or are dramatic (last sentence of stimulus).

Correct Answer: (D)


21. Question Type: Necessary Assumption

Gap: The argument suggests that because trees aren't as effective as native grasses when it comes to capturing carbon dioxide, the government incentives encouraging farmers to plant trees are actually worsening the problem of global warming. However, we don't necessarily know that the land where farmers are planting the trees could've been used for native grasses instead, or that native grasses were there already. What if native grasses couldn't be grown in that region for some reason? If this were the case, then the trees may actually be helping by capturing some of the carbon dioxide.

(A) This would certainly strengthen the argument that trees are making the problem worse, but this doesn't actually *need* to be the case. The argument never requires that trees actually emit carbon dioxide - just that they capture less carbon dioxide than do the native grasses.

(B) The first sentence of the stimulus tells us that the government incentives are *causing* many farmers to plant trees, but it doesn't need to be the case that government incentives are *required* in order for most farmers to plant trees.

(C) The stimulus never tells us anything about land that's been deforested - nothing needs to be the case about such land.

(D) See above. In order for it to be the case that the government incentives are worsening global warming (relatively speaking), it *must* be the case that at least some of the land where farmers are planting trees for that government money would've otherwise been used for native grasses (which would've captured even more carbon dioxide).

(E) We're only concerned with countries where government incentives for *tree-planting* have been put into place. We're not concerned with incentives encouraging the growth of native grasses.

Correct Answer: (D)


22. Question Type: Role of the Statement

The argument's conclusion is the stimulus' first sentence, and all of the following statements serve as support for that conclusion, either directly or indirectly. The statement in question is a subsidiary (intermediate) conclusion. It supports the first sentence and is supported by the stimulus' final two sentences.

(A) It supports the conclusion - it's not the conclusion itself. The first sentence is.

(B) The argument doesn't show this to be inconsistent with any evidence - the evidence actually *supports* this statement.

(C) No observed phenomenon is mentioned in the stimulus - everything's abstract.

(D) This is evidence that supports the argument - the argument doesn't ever go against this statement.

(E) See above. It serves as evidence for the argument's main conclusion (first sentence).

Correct Answer: (E)


23. Question Type: Point at Issue

(A) The first two statements made by the physician suggest that the physician would agree with this statement, but the trampoline enthusiast never expresses an opinion about whether a *significant* number of people are injured using them.

(B) Neither the physician nor the trampoline enthusiast makes a statement about what the *main* source of trampoline-related injuries is.

(C) The physician doesn't even come close to expressing an opinion about whether the rate of injuries per use is declining. The trampoline enthusiast suggests that the rate of injuries per *home trampoline* is declining, but the enthusiast doesn't express any opinion on the number of injuries per trampoline *user*.

(D) This is tempting, but neither the physician nor the trampoline enthusiast actually expresses an opinion on whether professional supervision tends to lead to a decrease in related injuries. It's simply that the physician favors professional supervision and the trampoline enthusiast opposes it.

(E) The physician explicitly states in the final sentence that trampolines should only be used when there's professional supervision, so the physician would likely agree with this choice. The trampoline enthusiast explicitly states "I agree" immediately following that statement, indicating that the enthusiast would disagree with this choice.

Correct Answer: (E)


24. Question Type: Strengthen (Principle)

The editorial concludes that the coverage of the local politician's nephew was good journalism because it was accurate and there was significant interest. For this question, we're just looking for a generalized version of this argument, because if a generalized version of it (in the form of a principle) is valid, then the specific reasoning in the stimulus is further justified.

(A) This is a negation of the situation in the stimulus, so it's not relevant. We're concerned with journalism *not deserving to be criticized if it *does* provide what people want.

(B) This is along the lines of a negation of the situation in the stimulus, so it's not relevant. We're concerned with journalism *not* deserving to be criticized if it *doesn't* intentionally misrepresent the facts. We'd actually want to go one step further and say "if it's accurate," since the information in the coverage was accurate in the stimulus.

(C) See above. If we learned that being accurate and covering a subject of interest to many constitutes good journalism, then the editorial's conclusion that the coverage was good journalism is greatly strengthened, as it met both of those criteria.

(D) We're concerned with what's sufficient to guarantee that something constitutes good journalism, not with what good journalism does in general.

(E) This is along the lines of a negation of the situation in the stimulus, so it's not relevant. We're concerned with journalism that *does* satisfy curiosity and *does* provide information, not journalism that doesn't. Additionally, we'd want to go one step beyond saying that something can be *considered* good journalism to say that it *definitely* is good journalism, as that would further justify the reasoning in the stimulus.

Correct Answer: (C)


25. Question Type: Must Be True

Because the stimulus contains multiple conditional indicators ("if" and "all"), it's a good idea to diagram it to make some links between statements:

1st sentence: C or R -> PP
2nd sentence: Most WD PP -> Art 
3rd sentence: NOT C PP -> NOT WD PP
4th sentence: C PP -> SI

Since the 3rd and 4th sentences both talk about being comfortable, we can take the contrapositive of one of them in order to link them together. I'd recommend taking the contrapositive of the 3rd one in order to make NOT WD into WD. This way, we can make the statement entirely positive (by also changing UC into C - essentially making that variable positive as well). Doing this gets us:

3rd sentence contrapositive: WD PP -> Comf PP (if a public place is well-designed, then it's a comfortable public place)
4th sentence: Comf PP -> SI (if it's a comfortable public place, then it has a spacious interior)

Linked: WD PP -> Comf PP -> SI

This is the only true linking we can easily do with this information, so it makes sense to just run through the choices and see if anything matches with what we've got (while keeping the first 2 sentences in mind as well).

(A) This is along the lines of a logic reversal of the 4th sentence, confusing necessary and sufficient conditions (since we know that restaurants are public places).

(B) This is a confused combination of the 2nd sentence and the contrapositive of the 3rd, while transposing the "most" into the wrong place. We can't infer anything about most public places that feature artwork, as this is a category never discussed in the stimulus.

(C) This is also a misapplication of the word "most" from the 2nd sentence. We know that most well-designed public places in general feature artwork, but we don't necessarily know that this ratio of majority extends within the category of coffeehouses in particular. In other words, just because most well-designed public places feature artwork, this doesn't mean that a majority of each *type* of public place features artwork - we don't know that the ratio holds true within categories.

(D) This one's a bit tricky, but we know from the first sentence of the stimulus that coffeehouses and restaurants must be public places, and we know from our linking of the contrapositive of the 3rd and 4th sentences that any well-designed public place must have a spacious interior.

(E) This is a logic reversal of our linking of the contrapositive of the 3rd sentence and the 4th sentence. Just because well-designed public places have spacious interiors, this doesn't guarantee that anything with a spacious interior is a well-designed public place (and we certainly don't know that this applies specifically to coffeehouses, either).

Correct Answer: (D)


Get the full LSAT PrepTest explanations for LSAT PrepTest 65 (and TONS of other exams) HERE.